Suppose 42% of the population has myopia. If a random sample of size 442 is selected, what is the probability that the proportion of persons with myopia will differ from the population proportion by less than 3%

Answers

Answer 1

Answer:

0.7994 = 79.94% probability that the proportion of persons with myopia will differ from the population proportion by less than 3%.

Step-by-step explanation:

To solve this question, we need to understand the normal probability distribution and the central limit theorem.

Normal Probability Distribution

Problems of normal distributions can be solved using the z-score formula.

In a set with mean [tex]\mu[/tex] and standard deviation [tex]\sigma[/tex], the z-score of a measure X is given by:

[tex]Z = \frac{X - \mu}{\sigma}[/tex]

The Z-score measures how many standard deviations the measure is from the mean. After finding the Z-score, we look at the z-score table and find the p-value associated with this z-score. This p-value is the probability that the value of the measure is smaller than X, that is, the percentile of X. Subtracting 1 by the p-value, we get the probability that the value of the measure is greater than X.

Central Limit Theorem

The Central Limit Theorem establishes that, for a normally distributed random variable X, with mean [tex]\mu[/tex] and standard deviation [tex]\sigma[/tex], the sampling distribution of the sample means with size n can be approximated to a normal distribution with mean [tex]\mu[/tex] and standard deviation [tex]s = \frac{\sigma}{\sqrt{n}}[/tex].

For a skewed variable, the Central Limit Theorem can also be applied, as long as n is at least 30.

For a proportion p in a sample of size n, the sampling distribution of the sample proportion will be approximately normal with mean [tex]\mu = p[/tex] and standard deviation [tex]s = \sqrt{\frac{p(1-p)}{n}}[/tex]

Suppose 42% of the population has myopia.

This means that [tex]p = 0.42[/tex]

Random sample of size 442 is selected

This means that [tex]n = 442[/tex]

Mean and standard deviation:

[tex]\mu = p = 0.42[/tex]

[tex]s = \sqrt{\frac{p(1-p)}{n}} = \sqrt{\frac{0.42*0.58}{442}} = 0.0235[/tex]

What is the probability that the proportion of persons with myopia will differ from the population proportion by less than 3%?

Proportion between 0.42 + 0.03 = 0.45 and 0.42 - 0.03 = 0.39, which is the p-value of Z when X = 0.45 subtracted by the p-value of Z when X = 0.39.

X = 0.45

[tex]Z = \frac{X - \mu}{\sigma}[/tex]

By the Central Limit Theorem

[tex]Z = \frac{X - \mu}{s}[/tex]

[tex]Z = \frac{0.45 - 0.42}{0.0235}[/tex]

[tex]Z = 1.28[/tex]

[tex]Z = 1.28[/tex] has a p-value of 0.8997

X = 0.39

[tex]Z = \frac{X - \mu}{s}[/tex]

[tex]Z = \frac{0.39 - 0.42}{0.0235}[/tex]

[tex]Z = -1.28[/tex]

[tex]Z = -1.28[/tex] has a p-value of 0.1003

0.8997 - 0.1003 = 0.7994

0.7994 = 79.94% probability that the proportion of persons with myopia will differ from the population proportion by less than 3%.


Related Questions

Urgent help!!!
*Picture included

Answers

Answer:

3x+4

Step-by-step explanation:

When you factor 9x^2+24x+16, it factors to (3x+4)^2

Factoring 9x^2 - 16 factors to (3x+4)(3x-4)

Therefore the common factor is 3x+4

I hope this helps!

In ABC, if CB AC≅ , m∠A = 3x + 18, m∠B = 7x – 58, and m∠C = 2x – 8, find x and the measure of each angle.

Answers

9514 1404 393

Answer:

  x = 19

  A = 30°

  B = C = 75°

Step-by-step explanation:

In an isosceles triangle, the angles opposite the congruent sides have the same measures.

  A = B

  3x +18 = 7x -58

  76 = 4x . . . . . . . . add 58-4x

  19 = x . . . . . . . . . divide by 4

Then the equal angles measure ...

  A = B = 3(19) +18 = 75

  C = 2(19) -8 = 30

Angles A, B, C measure 75°, 75°, 30°, respectively.

_____

Alternate solution

The sum of angles in a triangle is 180°, so you could write ...

  (3x +18) +(7x -58) +(2x -8) = 180

  12x = 228 . . . . . add 48

  x = 19 . . . . . divide by 12

There are four different colored balls in a bag. There is equal probability of selecting the red, black, green, or blue ball.What is the expected value of getting a green ball out of 20 experiments with replacement?

Answers

Answer:

The expected value is of 5 green balls.

Step-by-step explanation:

For each experiment, there are only two possible outcomes. Either it is a green ball, or it is not. Since there is replacement, the probability of a green ball being taken in an experiment is independent of any other experiments, which means that the binomial probability distribution is used to solve this question.

Binomial probability distribution

Probability of exactly x successes on n repeated trials, with p probability.

The expected value of the binomial distribution is:

[tex]E(X) = np[/tex]

20 experiments

This means that [tex]n = 20[/tex]

There is equal probability of selecting the red, black, green, or blue ball.

This means that 1 in 4 are green, so [tex]p = \frac{1}{4} = 0.25[/tex]

What is the expected value of getting a green ball out of 20 experiments with replacement?

[tex]E(X) = np = 20*0.25 = 5[/tex]

The expected value is of 5 green balls.

The expected value of getting a green ball out of 20 experiments with replacement is 5.

What is a binomial distribution?

The binomial probability distribution of the number of successes in a sequence of n independent experiments is the binomial distribution with parameters n and p.

As it is given that the probability of all the balls coming out of the bag is equal. Therefore, the probability of a green ball coming can be written as,

[tex]\text{Probability of Green Ball} = 0.25[/tex]

Also, we can write the probability of not getting a green ball can also be written as,

[tex]\rm Probability(\text{Not coming Green Ball}) = P(Red\ ball)+P(Black\ ball)+P(Blue\ ball)[/tex]

                                                         [tex]=0.25+0.25+0.25\\\\=0.75[/tex]

Now, as there are only two outcomes possible, therefore, the distribution of the probability is a binomial distribution. And we know that the expected value of a binomial distribution is given as,

[tex]\rm Expected\ Value, E(x) = np[/tex]

where n is the number of trials while p represents the probability.

Now, substituting the values, we will get the expected value,

[tex]\rm Expected\ Value, E(Green\ ball) = 20 \times 0.25 = 5[/tex]

Hence, the expected value of getting a green ball out of 20 experiments with replacement is 5.

Learn more about Binomial Distribution:

https://brainly.com/question/12734585

Which of the following best describes the relationship between angle a and angle bin the image below?

Answers

They are adjacent angles and linear pairs.
The are adjacent angles and linear pairs

At the Fidelity Credit Union, a mean of 5.8 customers arrive hourly at the drive-through window. What is the probability that, in any hour, more than 5 customers will arrive

Answers

Answer:

0.5217

Step-by-step explanation:

P(more than 5 customer arrive):

P(X>=6)=1-P(X<=5)=  1-∑x=0x e-λ*λx/x!= 0.5217

Solve for y.
5y – 10 = 10
y = [?]
What is y?

Answers

Answer:

y = [ 4 ]

Step-by-step explanation:

5y - 10 = 10

     +10  +10

5y = 20

/5     /5

y = 4

hope this helps ! ^^

Answer:

[tex]5y-10=10[/tex]

[tex]Add ~10[/tex]

[tex]5y=10+10[/tex]

[tex]5y=20[/tex]

[tex]divide ~by ~5[/tex]

[tex]y=4[/tex]

[tex]ANSWER: y=4[/tex]

-----------------------------

HOPE IT HELPS

HAVE A GREAT DAY!!

find the place value of 1 in 382619.​

Answers

Answer:

Place value of 1 = 1 × 10 = 10

Step-by-step explanation:

In 382619,

Place of 1 = Tens

Place value of 1 = 1 × 10 = 10

The place value of 1 is 10 as it is in the tens column.

Giving a test to a group of students, the grades and gender are summarized below

A B C Total
Male 20 10 13 43
Female 15 2 11 28
Total 35 12 24 71


If one student is chosen at random,

Find the probability that the student did NOT get an "B"

Answers

Answer:

59 / 71

Step-by-step explanation:

Given the data :

A B C Total

Male 20 10 13 43

Female 15 2 11 28

Total 35 12 24 71

The probability of randomly selecting a Student that got B ;

Probability = required outcome / Total possible outcomes

P(getting B) = number of students who got B / total number of students

P(getting B) = 12 / 71

Probability of getting B = 12 /71

Probability of not getting B = P(getting B)' = 1 - P(getting B)

Probability that student did not get "B" = 1 - 12/71 = 59 / 71

Which of the following is NOT true of a perpendicular bisect or?

Answers

Answer:

The forth option

It forms a right angle with the segment.

I need help solving this problem. Thanks

Answers

9514 1404 393

Answer:

  f = 2T/(v1 +v2)

Step-by-step explanation:

Multiply by the inverse of the coefficient of f.

  [tex]T=f\cdot\dfrac{v_1+v_2}{2}\\\\f=\dfrac{2T}{v_1+v_2}[/tex]

What is center of a circle whose equation is x2

Answers

Answer:

I think it is 160 x2 so you would probably divide 160 by x2 which would 144

Step-by-step explanation:

PLEASE ANSWER MY QUESTION AND EXPLAIN RIGHT

Answers

Answer:

$ 1943

Step-by-step explanation:

You two congruent trapezoids.

Find the area of one and multiply by 2.

A = [tex]\frac{base_{1} + base_{2} }{2}[/tex] x  h

  = [tex]\frac{28+39}{2}[/tex] x 14.5

  = [tex]\frac{67}{2}[/tex] x 14.5

  = 33.5 x 14.5

  = 485.75

  = 485.75 x 2 (Two trapezoids)

  = 971.50

  = 971.50 x 2 (two dollars a square foot)

  = 1943.00

find the value of...​

Answers

Answer:

1

Step-by-step explanation:

tan(1)tan(2)....tan(89)=?

Recall tan(90-x)=cot(x) and cot(x)tan(x)=1.

tan(89)=tan(90-1)=cot(1)

tan(88)=tan(90-2)=cot(2)

tan(87)=tan(90-3)=cot(3)

...

tan(46)=tan(90-44)=cot(44)

tan(45)=tan(90-45)=cot(45)

So we can replace the last half of the factors with cotangent of the angles in the first half.

The only one that doesn't get a partner is the exact middle factor which is tan(45).

So this is whar we have:

tan(1)tan(2)tan(3)....tan(45)....cot(3)cot(2)cot(1)

So you should see that cot(1)tan(1)=1 and cot(2)tan(2)=1 and so on....

So the product equals tan(45) and tan(45)=1 using unit circle.

Which of the following expressions has a Value of 6.18???

Answers

Answer:

B. -21.012÷ -3.4

its yr correct ans.

hope it helps

stay safe healthy and happy.
B) -21.012/-3.4= 6.18

In what ratio of line x-y-2=0 divides the line segment joining (3,-1) and (8,9)?​

Answers

Let the given points ( 3 , -1 ) and ( 8 , 9 ) be A and B respectively. Let A ( 3 , - 1 ) be ( x₁ , y₁ ) and B ( 8 , 9 ) be ( x₂ , y₂ ). Let the point P ( x , y ) divides the line segment of joining points A ( 3 , -1 ) and ( 8 , 9 ) in the ratio m : n. Let m be m₁ and n be m₂ We know that :

[tex] \large{ \tt{❁ \: USING \: INTERNAL \: SECTION \: FORMULA: }}[/tex]

[tex] \large{ \bf{✾ \: P(x \:, y \: ) = ( \frac{m_{1}x_{2} + m_{2}x_{1}}{m_{1} + m_{2}} \: ,\: \frac{m_{1}y_{2} + m_{2}y_{1}}{m_{1} + m_{2}}) }}[/tex]

[tex] \large{ \bf{⟹ \: ( \frac{8m + 3n}{m + n} , \: \frac{9m -n}{m + n}) }}[/tex]

Since point P lies on the line x - y - 2 = 0 ,

[tex] \large{ \bf{ ⟼\frac{8m + 3n}{m + n} - \frac{9m - n}{m + n} - 2 = 0 }}[/tex]

[tex] \large{ \bf{⟼ \: \frac{8m + 3n - 9m + n}{m + n} - 2 = 0 }}[/tex]

[tex] \large{ \bf{⟼ \: \frac{4n - m}{ m + n} - 2 = 0 }}[/tex]

[tex] \large{⟼ \: \bf{ \frac{4n - m}{m + n }} = 2} [/tex]

[tex] \large{ \bf{⟼ \: 4n - m = 2m + 2n}}[/tex]

[tex] \large{ \bf{⟼ \: 4n -2 n = 2m + m}}[/tex]

[tex] \large{ \bf{⟼2n = 3m}}[/tex]

[tex] \large{ \bf{⟼ \: 3m = 2n}}[/tex]

[tex] \large{ \bf{⟼ \: \frac{m}{n} = \frac{2}{3} }}[/tex]

[tex] \boxed{ \large{ \bf{⟼ \: m : \: n = 2: \: }3}}[/tex]

Hence , The required ratio is 2 : 3 .

-Hope I helped! Let me know if you have any questions regarding my answer and also notify me , if you need any other help! :)

▁▁▁▁▁▁▁▁▁▁▁▁▁▁▁▁▁

Which choice is equivalent to √3 *√8*√5
A. 2√30
B. 4√30
C. 10√12
D. 24√5

Answers

[tex]\implies {\blue {\boxed {\boxed {\purple {\sf { A. \:2 \sqrt{30} }}}}}}[/tex]

[tex]\sf \bf {\boxed {\mathbb {STEP-BY-STEP\:EXPLANATION:}}}[/tex]

[tex] = \sqrt{3} \times \sqrt{8} \times \sqrt{5} [/tex]

[tex] = \sqrt{3 \times 2 \times 2 \times 2 \times 5} [/tex]

[tex] = \sqrt{ ({2})^{2} \times 2 \times 3 \times 5} [/tex]

[tex] = 2 \sqrt{2 \times 3\times 5} [/tex]

[tex] = 2 \sqrt{30} [/tex]

Note:

[tex] \sqrt{ ({a})^{2} } = a[/tex]

[tex]\bold{ \green{ \star{ \orange{Mystique35}}}}⋆[/tex]

Answer:

A. 2√30

Step-by-step explanation:

[tex] \small \sf \: \sqrt{3} \times \sqrt{8} \times \sqrt{5} \\ [/tex]

split √8

[tex] \small \sf \leadsto \sqrt{3 × 2 × 2 × 2 × 5} [/tex]

[tex] \small \sf \leadsto \: 2 \sqrt{2 \times 3 \times 5} [/tex]

[tex] \small \sf \leadsto \: 2 \sqrt{30} [/tex]

Which statement is true about quadrilateral ABCD with vertices A(2, 8), B(3, 11), C(4, 8), and D(3, 5)?

Answers

Answer:

The quadrilateral is a rhombus

Step-by-step explanation:

Given

[tex]A = (2, 8)[/tex]

[tex]B = (3, 11)[/tex]

[tex]C = (4, 8)[/tex]

[tex]D=(3, 5)[/tex]

Required

The true statement

Calculate slope (m) using

[tex]m = \frac{y_2 - y_1}{x_2 - x_1}[/tex]

Calculate distance using:

[tex]d= \sqrt{(x_2 - x_1)^2 + (y_2 -y_1)^2}[/tex]

Calculate slope and distance AB

[tex]m_{AB} = \frac{11 - 8}{3 - 2}[/tex]

[tex]m_{AB} = \frac{3}{1}[/tex]

[tex]m_{AB} = 3[/tex] -- slope

[tex]d_{AB}= \sqrt{(3 - 2)^2 + (11 -8)^2}[/tex]

[tex]d_{AB}= \sqrt{10}[/tex] -- distance

Calculate slope and distance BC

[tex]m_{BC} = \frac{8 - 11}{4 - 3}[/tex]

[tex]m_{BC} = \frac{- 3}{1}[/tex]

[tex]m_{BC} = -3[/tex] -- slope

[tex]d_{BC} = \sqrt{(4-3)^2+(8-11)^2[/tex]

[tex]d_{BC} = \sqrt{10}[/tex] --- distance

Calculate slope CD

[tex]m_{CD} = \frac{5 - 8}{3 - 4}[/tex]

[tex]m_{CD} = \frac{- 3}{- 1}[/tex]

[tex]m_{CD} = 3[/tex] -- slope

[tex]d_{CD} = \sqrt{(3-4)^2+(5-8)^2}[/tex]

[tex]d_{CD} = \sqrt{10}[/tex] -- distance

Calculate slope DA

[tex]m_{DA} = \frac{8 - 5}{2 - 3}[/tex]

[tex]m_{DA} = \frac{3}{- 1}[/tex]

[tex]m_{DA} = -3[/tex] -- slope

[tex]d_{DA} = \sqrt{(2-3)^2 + (8-5)^2}[/tex]

[tex]d_{DA} = \sqrt{10}[/tex]

From the computations above, we can see that all 4 sides are equal, i.e. [tex]\sqrt{10}[/tex]

And the slope of adjacent sides are negative reciprocal, i.e.

[tex]m_{AB} = 3[/tex]  and [tex]m_{CD} = -3[/tex]

[tex]m_{CD} = 3[/tex] and [tex]m_{DA} = -3[/tex]

The quadrilateral is a rhombus

You want to be able to withdraw the specified amount periodically from a payout annuity with the given terms. Find how much the account needs to hold to make this possible. Round your answer to the nearest dollar. Regular withdrawal: $1200 Interest rate: 2.5% Frequency monthly Time: 26 years
what is the account balance?​

Answers

Step-by-step explanation:

principal=?. interest=$1200. rate =2. 5%. time=26 NOW, principal=I×100/T×R= $1200×100/26×2. 5=1846. 15

9514 1404 393

Answer:

  $275,098.25

Step-by-step explanation:

The principal amount can be found using the annuity formula.

 A = P(r/12)/(1 - (1 +r/12)^(-12t))

where A is the monthly payment, P is the principal amount, r is the annual interest rate, and t is the number of years.

Solving for P, we have ...

  P = A(12/r)(1 -(1 +r/12)^(-12t)) = 1200(12/0.025)(1 -(1 +.025/12)^(-12·26))

  = $275,098.25

The account balance needs to be $275,098.25.

Using the simple spinner below what is the probability of landing on either 2, 4, or 7?

Answers

Answer:

3/10

Step-by-step explanation:

Total possibilities = 10

favourable possibilities = 3

Answer:

A

Step-by-step explanation:

There is a 1 out of 10 chance that it will land on 2.

There is a 1 out of 10 chance that it will land on 4.

There is a 1 out of 10 chance that it will land on 7.

[tex]\frac{1}{10}\cdot3=\frac{3}{10}[/tex] so the anwser is A.

Can you please help me solve this step by step?

Answers

Answer:

2/3

Step-by-step explanation:

[tex]2 \frac{1}{4} : \frac{1}{2}[/tex] = [tex]\frac{9}{4} : \frac{1}{2}[/tex]

[tex]\frac{\frac{9}{4} }{\frac{1}{2} }[/tex] = [tex]\frac{3}{x}[/tex]

3 * 1/2 = 9/4x

3/2 = 9/4 x

x = 3/2 ÷ 9/4 = 3/2 * 4/9 = 12/18 = 6/9 = 2/3

Okay, let's calculate the year end adjustment for overhead. Based on the data below, determine the amount of the year end adjustment to cost of goods sold due to over or under allocated manufacturing overhead during the year

Answers

Answer:

the adjustment made to the cost of goods sold is -$2,014

Step-by-step explanation:

The computation of the adjustment made to the cost of goods sold is given below:

Total actual overhead expenses $110,822

Less: Total overheads allocated -$112,836

Adjustment made to the cost of goods sold -$2,014

Hence, the adjustment made to the cost of goods sold is -$2,014

The same should be considered

ok i think you guys can do it​

Answers

[tex] {64}^{ \frac{2}{3} } \div {27}^{ \frac{5}{3} } \times 54 \\ = > \: {({2}^{3} )}^{ \frac{2}{3} } \div ({{3}^{3}})^{ \frac{5}{3} } \times 54 \\ = > \: {2}^{2} \div {3}^{5} \times 54 \\ = > \: 4 \div 243 \times 54 \\ = > \: 4 \div 13122 \\ = > \: \frac{4}{13122} \\ = > \: \frac{2}{6561} [/tex]

Hope it helps!!!!!!!!!!

((64)^1/3)^2 / ((27)^1/3)^5 x 54
= 4^2 / 3^5 x 54
= 16 x 54 / 243 [= 16 x (2 x 27) / (9 x 27)]
= 32/9

Which proportion resulted in the equation 3a = 7b?

StartFraction 3 over a EndFraction = StartFraction 7 over b EndFraction
StartFraction 3 over b EndFraction = StartFraction 7 over a EndFraction
StartFraction a over b EndFraction = StartFraction 3 over 7 EndFraction
StartFraction 3 over 7 EndFraction = StartFraction 3 over b EndFraction

Answers

Answer:

The correct one is 3 over b equals 7 over a

Answer:

3/b = 7/a

Step-by-step explanation:

I took it on Edge

Two cell phone companies charge a flat fee plus an added cost for each minute or part of a minute used. The cost is represented by C and the number of minutes is represented by t.
Call-More: C = 0.40t + 25 Talk-Now: C = 0.15t + 40

Answers

Answer:

The call more is cheaper than talk-now.

Step-by-step explanation:

The companies charge a flat fee plus an added cost for each minute or part of a minute used for two companies are as follows :

Call-More: C = 0.40t + 25 Talk-Now: C = 0.15t + 40

We need to find which company is cheaper if a customer talks for 50 minutes.

For call more,

C = 0.40(50) + 25 = 45 units

For talk-now,

C = 0.15(50) + 40 = 47.5 units

So, it can be seen that call more is cheaper than talk-now.

SCALCET8 3.9.017.MI. Two cars start moving from the same point. One travels south at 48 mi/h and the other travels west at 20 mi/h. At what rate is the distance between the cars increasing two hours later

Answers

Answer:

The rate at which the distance between the cars increasing two hours later=52mi/h

Step-by-step explanation:

Let

Speed of one car, x'=48 mi/h

Speed of other car, y'=20 mi/h

We have to find the rate at which the distance between the cars increasing two hours later.

After 2 hours,

Distance traveled by one car

[tex]x=48\times 2=96 mi[/tex]

Using the formula

[tex]Distance=Time\times speed[/tex]

Distance traveled by other car

[tex]y=20\times 2=40 mi[/tex]

Let z be the distance between two cars after 2 hours later

[tex]z=\sqrt{x^2+y^2}[/tex]

Substitute the values

[tex]z=\sqrt{(96)^2+(40)^2}[/tex]

z=104 mi

Now,

[tex]z^2=x^2+y^2[/tex]

Differentiate w.r.t t

[tex]2z\frac{dz}{dt}=2x\frac{dx}{dt}+2y\frac{dy}{dt}[/tex]

[tex]z\frac{dz}{dt}=x\frac{dx}{dt}+y\frac{dy}{dt}[/tex]

Substitute the values

[tex]104\frac{dz}{dt}=96\times 48+40\times 20[/tex]

[tex]\frac{dz}{dt}=\frac{96\times 48+40\times 20}{104}[/tex]

[tex]\frac{dz}{dt}=52mi/h[/tex]

Hence, the rate at which the distance between the cars increasing two hours later=52mi/h

rotation 180 degrees about the origin.​

Answers

Answer:

Click the rotate 'button' twice.

Observe.

The rotate button is rotating the image about the orgin.

Answer:

Click the rotate 'button' twice.

Observe.

The rotate button is rotating the image about the orgin.

Step-by-step explanation:

How do you solve this problem and what did you do to gain the answer 1/64+5/8-3/32=?

Answers

the answer is 35/64 or 0.546875 in decimal form

first we write all numerators above the least common denominator 64
1+40-6
————
64

next we solve the numerators by doing 1+40-6 to get 35/64

Answer:

the answer is 35/64(in fraction) but in decimals it's 0.55

A boy had 3 apples and lost one, how many does he have now

Answers

Step-by-step explanation:

i would love to say 2 but the word had shows that he does not have 3 apples anymore so the answer is either

0 or -1

The number of apples left after taking the 1 apple from 3 apples by a person is 2 apples.

What is subtraction?

Subtraction stands for the resultant number, which exists acquired by taking the difference of a number from another number.

Let a number be subtracted from the number b. Then the consequent number after subtracting b from a will be,

d = b - a

Here, (a, b) exists the real numbers.

It exists given that there exist 3 apples. 1 apple stand was taken. Let's assume after taking the 3 apples, that there exist x apples remaining.

As there exist a total of 3 apples and 1 apple stand taken, then to estimate the number of apples left, we must subtract 1 apple from 3 apples.

Therefore, the total apples left exist,

x = 3 - 1

x = 2

To learn more about subtraction operation

https://brainly.com/question/26883387

#SPJ2

Someone help me please

Answers

9514 1404 393

Answer:

A = (0, 1)B = (3, -2)area = 4.5 square units

Step-by-step explanation:

Rewriting the equations to make x the subject, we have ...

  x = y² -1 . . . . . [eq1]

  x = 1 - y . . . . . .[eq2]

At the points of intersection, the difference will be zero.

  y² -1 -(1 -y) = 0

  y² +y -2 = 0

  (y -1)(y +2) = 0

The y-coordinates of points A and B are 1 and -2.

The corresponding x-coordinates are ...

  x = 1 -{1, -2} = {1 -1, 1+2} = {0, 3}

Then A = (0, 1) and B = (3, -2).

__

A differential of area can be written ...

  (x2 -x1)dy = ((1 -y) -(y² -1))dy = (2 -y -y²)dy

Integrating this over the interval y = [-2, 1] gives the area.

  [tex]\displaystyle A=\int_{-2}^1(2-y-y^2)\,dy=\left.(2y-\dfrac{1}{2}y^2-\dfrac{1}{3}y^3)\right|_{-2}^1\\\\=\left(2-\dfrac{1}{2}-\dfrac{1}{3}\right)-\left(2(-2)-\dfrac{(-2)^2}{2}-\dfrac{(-2)^3}{3}\right)=\dfrac{7}{6}+4+2-\dfrac{8}{3}\\\\=\boxed{4.5}[/tex]

The area of the shaded region is 4.5 square units.

In rural Ireland, a century ago, the students had to form a line. The student at the front of the line would be asked to spell a word. If he spelled it correctly, he was allowed to sit down. If not, he received a whack on the hand with a switch and was sent to the end of the line. Suppose the student could spell correctly 60% of the words in the lesson. What is the probability that the student would be able to sit down before receiving four whacks in the hand?

Answers

Answer:

The answer is "0.9102"

Step-by-step explanation:

P(student spell correct) = 0.6

P(student spell incorrect)=1-0.6=0.4

X=the pupil will be allowed to sit down after receiving three slaps on the hand Thus, X would assume Value

X=0 (student sit sans getting whacks)

X=1 (student sit down without receiving 1 whack) (student sit down without receiving 1 whack)

X=2 (student take a seat before receiving 2 whacks)

X=3 (student sit down after receiving 3 punches)

[tex]\to P(X)=0.6 \times 0.4^0 +0.6 \times 0.4 + 0.6 \times 0.4 ^2 + 0.6 \times 0.3^3[/tex]

             [tex]=0.6 \times 1+0.24 + 0.6 \times 0.09 + 0.6 \times 0.027\\\\=0.6 +0.24 + 0.054 + 0.0162\\\\=0.9102\\\\[/tex]

Other Questions
What is the coefficient of x^8 in the expansion of (x+4)^12 ? Can someone help me with this question and show me how to solve them pleases? Question 14 (5 points)Which of the following is equivalent to cos (23)?cos (90)sin (67)sin (23)cos (67) Lesson: Hierarchical Taxonomic System of ClassificationMELCS: 1. Explain the concept of a species.2. Classify organisms using the hierarchical taxonomic system.1. Directions: Using the Linnaean system of classification, describe these living things.CATEGORYHumanDomainKingdomPhylumClassOrderFamilyGenusSpeciesW How do Americans influence the government? Characteristics of social consciousness in Vietnam Integrate with respect to x-2/3((2-5x)^3) Leahy Corp. sells $300,000 of bonds to private investors. The bonds are due in five years, have a 6% coupon rate, and interest is paid semiannually. The bonds were sold to yield 4%. Group of answer choices The bonds were sold at a discount, with annual interest expenses more than $18,000 The bonds were sold at a discount, with annual interest expenses less than $18,000 The bonds were sold at a premium, with annual interest expenses more than $18,000 The bonds were sold at a premium, with annual interest expenses less than $18,000 The bonds were sold at par, with annual interest expenses equal to $18,000 What is the minimum angular spread (in rad) of a 534 nm wavelength manganese vapor laser beam that is originally 1.19 mm in diameter Suppose an enzyme and its substrate obey the lock and key model of enzyme catalysis. Which of the following would be true of the enzyme? a. only one substrate could be converted to product by the enzyme b. the active site of the enzyme must be rigid c. the entire enzyme must be rigid d. the active site of the enzyme must be flexible e. the enzyme could bind different substrates if the substrates shared a common motif somewhere in their structures Mike wants to find out whether campers prefer cotton or nylon sleeping bags. He asks a group of female campers to try both nylon and cotton sleeping bags. The sleeping bags were rated for softness and warmth. Mike intends to generalize the findings to apply to all campers. What is the confounding variable in Mike's experiment Help me find the mistake in this sentence The branch of life sciences which involves the structure and function of the brain and nervous system, including neurons, nerves, and nervous tissue is called Which Group is in the second column of the periodic table?O A. Noble gasesO B. Alkaline earth metalsO C. Alkali metalsO D. Halogens If interest is 8% and it is compounded semiannually, and after one year, the total value is $10,816, what was the original investment? Which of the following statement is true? a) Consideration need not be any particular value. b) Consideration should accord the deed of the person who is at the position to do it. c) Consideration may proceed the promise that the promise or promisor pass to each other. d) An act done at the desire of a third party is equal to consideration. I had block the instructions but basically it said State what additional information is required in order to know that the triangles in the image below are congruent for the reason given. Who is the intended audience for his present ion? Instructions: Solve the following linearequation4(n + 5) 2(5 + 7n) = -70n = The _ is made up of glands found throughout the body that release hormones into the bloodstream Although Tamara seems to be of average weight and height, her roommate notices that she has some potentially unhealthy eating behaviors. A few days after they go grocery shopping, many of the high calorie, rich foods turn up missing. Her roommate also notices that Tamara spends hours upon hours at the gym and often takes laxatives. Tamara's roommate is concerned that Tamara may be struggling with _____